Re: [obm-l] Ajuda numa desigualdade.

2016-01-28 Por tôpico Esdras Muniz
L = ((1+1/(n+1))^(n+1))/(1+1/n)^n = ((1 - 1/(n+1)²)^n)((n+2)/(n+1))

Use que (1 - x)^n > 1 - nx, Para x \in (0, 1)

L > (1 - n/(n+1)²)((n+2)/(n+1)) = ((n²+n+1)/(n²+2n+1))((n+2)/(n+1))
= (n³+3n²+3n+2)/(n³+3n²+3n+1) > 1.



Esse último termo é maior que 1.

Em 28 de janeiro de 2016 09:41, Douglas Oliveira de Lima <
profdouglaso.del...@gmail.com> escreveu:

> Opa Marcelo, muito obrigado mesmo, eu estou procurando uma solução
> daquelas tipo
> desigualdades, onde efetuamos uma estratégia para chegar no resultado,
> tipo uma daquelas que tu encontra no livro de combinatória do MOrgado(o
> problema das apostas).
> Mas valeu, se conseguir uma dessas me manda novamente por favor.
> Abraço
> Douglas Oliveira
>
> Em 28 de janeiro de 2016 01:26, Marcelo Salhab Brogliato <
> msbro...@gmail.com> escreveu:
>
>> Oi, Douglas, tudo bem?
>>
>> Se provarmos que f(x) = (1 + 1/x)^x é estritamente crescente, então está
>> provada sua desigualdade.
>>
>> Uma maneira é fazer isso usando cálculo. Seja g(x) = ln(f(x)) = x ln(1 +
>> 1/x). Assim, se provarmos que g(x) é estritamente crescente, então f(x)
>> também será (exercício: prove essa afirmação).
>>
>> g'(x) = ln(1 + 1/x) + x * (-1/x^2) / (1 + 1/x)  = ln(1 + 1/x) - (1/x) /
>> (1 + 1/x) = ln(1 + 1/x) - 1/(1+x)
>>
>> Temos que mostrar que g'(x) > 0 para todo x.
>>
>> Sabemos que ln(x) < x - 1, para x != 1. Aplicando essa desigualdade em
>> 1/x, temos: ln(1/x) < 1/x - 1 => ln(x) > 1 - 1/x, para x != 1.
>>
>> Aplicando a desigualdade acima em 1+1/x, temos: ln(1+1/x) > 1 - 1/(1 +
>> 1/x) = (1/x) / (1 + 1/x) = 1/(1+x). Logo: ln(1+1/x) > 1/(1+x) => g'(x) > 0
>> para todo x (já que 1+1/x > 1).
>>
>> Abraços,
>> Salhab
>>
>> 2016-01-28 0:34 GMT-02:00 Douglas Oliveira de Lima <
>> profdouglaso.del...@gmail.com>:
>>
>>> Olá caros amigos, gostaria de uma ajuda na seguinte desigualdade
>>> (1+1/n)^n<(1+1/n+1)^(n+1), para n natural.
>>>
>>> Agradeço desde já.
>>>
>>>
>>
>


-- 
Esdras Muniz Mota
Mestrando em Matemática
Universidade Federal do Ceará


RE: [obm-l] Ajuda numa desigualdade.

2016-01-28 Por tôpico Luís
Sauda,c~oes, oi Douglas, 
Vou dar uma dica: faça a_(n+1) = ? e a_1=……=a_n = ?? Dai use G  <= A ( no caso 
G  < A ) .
Abs, L. 

Date: Thu, 28 Jan 2016 16:15:11 -0200
Subject: RE: [obm-l] Ajuda numa desigualdade.
From: profdouglaso.del...@gmail.com
To: obm-l@mat.puc-rio.br

Erro? Bom no meu celular acho que saiu as fórmulas todas fora de ordem rs
Em 28/01/2016 16:02, "Bruno Lira"  escreveu:









Primeiramente, tome
a função logaritmântica f(x) = ln(x) cujo domínio é o conjuntos dos
números reais maiores que ou igual a zero. Note que a função f é
injetora. Portanto, para provarmos que:




n n+1
( 1 + 1
)<  ( 1 +   1  
 )
(   n ) (
   n+1 )




basta provar que:




   (n)   (  n+1)
ln(  ( 1 + 1
)   )   <  ln( ( 1 +   1 )  )
   ( (   n ) )   ( ( n+1)  ) .




De fato, temos que:




   (n)( n+1)
ln(  ( 1 + 1
)  )   –   ln(  ( 1 +   1   )   )=
   ( (  n )  )((  n + 1)   ) 





   (n)(n+1)
ln(  ( n
+ 1 )  )   –   ln( (
n +  2 
) )=
   ( (n)  
)( (  n + 1  )) 





   (   2n  )   

ln(  (   n
+ 1  )  .  n+1
) 
; Das
propriedades de logaritmo.
   ( (n (n+2))   n+2 ) 





Daí:




   ( n   ) 
ln(  (  n^2
+ 2n + 1  )  . 
n+1 )
 

   ( (n^2 + 2n   )n+2) 






Comon^2 + 2n < n^2 + 2n + 1en+1 < n + 2temos
que:





n
 

(  n^2 +
2n + 1 )  . 
n+1<1

(n^2 + 2n)   n+2
E da injetividade da função f temos:





   ( n   ) ln( ( n^2 + 2n + 1 ) . n+1 )   <
ln(1)=0   ( (n^2 + 2n   )n+2)
Isto é:

   (n)(n+1)ln( ( 1 + 1 )  ) – ln( ( 1 + 1 ) 
 )<0   ( (  n )  )( ( n+1  )  )






Logo,





n n+1( 1 + 1 ) < ( 1 + 1 )(   n ) ( n+1 )   
   C.Q.D
P.S.: Se tiver algum erro me avisem por favor.
From: esdrasmunizm...@gmail.com
Date: Thu, 28 Jan 2016 12:18:03 -0300
Subject: Re: [obm-l] Ajuda numa desigualdade.
To: obm-l@mat.puc-rio.br

L = ((1+1/(n+1))^(n+1))/(1+1/n)^n = ((1 - 1/(n+1)²)^n)((n+2)/(n+1)) 
Use que (1 - x)^n > 1 - nx, Para x \in (0, 1)
L > (1 - n/(n+1)²)((n+2)/(n+1)) = ((n²+n+1)/(n²+2n+1))((n+2)/(n+1)) = 
(n³+3n²+3n+2)/(n³+3n²+3n+1) > 1.


Esse último termo é maior que 1.
Em 28 de janeiro de 2016 09:41, Douglas Oliveira de Lima 
 escreveu:
Opa Marcelo, muito obrigado mesmo, eu estou procurando uma solução daquelas 
tipodesigualdades, onde efetuamos uma estratégia para chegar no resultado, tipo 
uma daquelas que tu encontra no livro de combinatória do MOrgado(o problema das 
apostas).Mas valeu, se conseguir uma dessas me manda novamente por 
favor.AbraçoDouglas Oliveira
Em 28 de janeiro de 2016 01:26, Marcelo Salhab Brogliato  
escreveu:
Oi, Douglas, tudo bem?
Se provarmos que f(x) = (1 + 1/x)^x é estritamente crescente, então está 
provada sua desigualdade.
Uma maneira é fazer isso usando cálculo. Seja g(x) = ln(f(x)) = x ln(1 + 1/x). 
Assim, se provarmos que g(x) é estritamente crescente, então f(x) também será 
(exercício: prove essa afirmação).
g'(x) = ln(1 + 1/x) + x * (-1/x^2) / (1 + 1/x)  = ln(1 + 1/x) - (1/x) / (1 + 
1/x) = ln(1 + 1/x) - 1/(1+x)
Temos que mostrar que g'(x) > 0 para todo x.
Sabemos que ln(x) < x - 1, para x != 1. Aplicando essa desigualdade em 1/x, 
temos: ln(1/x) < 1/x - 1 => ln(x) > 1 - 1/x, para x != 1.
Aplicando a desigualdade acima em 1+1/x, temos: ln(1+1/x) > 1 - 1/(1 + 1/x) = 
(1/x) / (1 + 1/x) = 1/(1+x). Logo: ln(1+1/x) > 1/(1+x) => g'(x) > 0 para todo x 
(já que 1+1/x > 1).
Abraços,Salhab
2016-01-28 0:34 GMT-02:00 Douglas Oliveira de Lima 
:
Olá caros amigos, gostaria de uma ajuda na seguinte desigualdade 
(1+1/n)^n<(1+1/n+1)^(n+1), para n natural.
Agradeço desde já.







-- 
Esdras Muniz Mota
Mestrando em Matemática
Universidade Federal do Ceará



  
  

Re: [obm-l] Ajuda numa desigualdade.

2016-01-28 Por tôpico victorcarlos
No livro do Yaglon de olimpíadas russas tem a solução. 
Abs
Carlos Victor

Enviado por Samsung Mobile

 Mensagem original De : Douglas Oliveira de 
Lima  Data:28/01/2016  00:34  
(GMT-03:00) Para: obm-l@mat.puc-rio.br Assunto: [obm-l] 
Ajuda numa desigualdade. 
Olá caros amigos, gostaria de uma ajuda na seguinte desigualdade 
(1+1/n)^n<(1+1/n+1)^(n+1), para n natural.

Agradeço desde já.



Re: [obm-l] Ajuda numa desigualdade.

2016-01-28 Por tôpico Douglas Oliveira de Lima
Opa Marcelo, muito obrigado mesmo, eu estou procurando uma solução daquelas
tipo
desigualdades, onde efetuamos uma estratégia para chegar no resultado, tipo
uma daquelas que tu encontra no livro de combinatória do MOrgado(o problema
das apostas).
Mas valeu, se conseguir uma dessas me manda novamente por favor.
Abraço
Douglas Oliveira

Em 28 de janeiro de 2016 01:26, Marcelo Salhab Brogliato  escreveu:

> Oi, Douglas, tudo bem?
>
> Se provarmos que f(x) = (1 + 1/x)^x é estritamente crescente, então está
> provada sua desigualdade.
>
> Uma maneira é fazer isso usando cálculo. Seja g(x) = ln(f(x)) = x ln(1 +
> 1/x). Assim, se provarmos que g(x) é estritamente crescente, então f(x)
> também será (exercício: prove essa afirmação).
>
> g'(x) = ln(1 + 1/x) + x * (-1/x^2) / (1 + 1/x)  = ln(1 + 1/x) - (1/x) / (1
> + 1/x) = ln(1 + 1/x) - 1/(1+x)
>
> Temos que mostrar que g'(x) > 0 para todo x.
>
> Sabemos que ln(x) < x - 1, para x != 1. Aplicando essa desigualdade em
> 1/x, temos: ln(1/x) < 1/x - 1 => ln(x) > 1 - 1/x, para x != 1.
>
> Aplicando a desigualdade acima em 1+1/x, temos: ln(1+1/x) > 1 - 1/(1 +
> 1/x) = (1/x) / (1 + 1/x) = 1/(1+x). Logo: ln(1+1/x) > 1/(1+x) => g'(x) > 0
> para todo x (já que 1+1/x > 1).
>
> Abraços,
> Salhab
>
> 2016-01-28 0:34 GMT-02:00 Douglas Oliveira de Lima <
> profdouglaso.del...@gmail.com>:
>
>> Olá caros amigos, gostaria de uma ajuda na seguinte desigualdade
>> (1+1/n)^n<(1+1/n+1)^(n+1), para n natural.
>>
>> Agradeço desde já.
>>
>>
>


RE: [obm-l] Ajuda numa desigualdade.

2016-01-28 Por tôpico Bruno Lira






Primeiramente, tome
a função logaritmântica f(x) = ln(x) cujo domínio é o conjuntos dos
números reais maiores que ou igual a zero. Note que a função f é
injetora. Portanto, para provarmos que:




n n+1
( 1 + 1
)<  ( 1 +   1  
 )
(   n ) (
   n+1 )




basta provar que:




   (n)   (  n+1)
ln(  ( 1 + 1
)   )   <  ln( ( 1 +   1 )  )
   ( (   n ) )   ( ( n+1)  ) .




De fato, temos que:




   (n)( n+1)
ln(  ( 1 + 1
)  )   –   ln(  ( 1 +   1   )   )=
   ( (  n )  )((  n + 1)   ) 





   (n)(n+1)
ln(  ( n
+ 1 )  )   –   ln( (
n +  2 
) )=
   ( (n)  
)( (  n + 1  )) 





   (   2n  )   

ln(  (   n
+ 1  )  .  n+1
) 
; Das
propriedades de logaritmo.
   ( (n (n+2))   n+2 ) 





Daí:




   ( n   ) 
ln(  (  n^2
+ 2n + 1  )  . 
n+1 )
 

   ( (n^2 + 2n   )n+2) 






Comon^2 + 2n < n^2 + 2n + 1en+1 < n + 2temos
que:





n
 

(  n^2 +
2n + 1 )  . 
n+1<1

(n^2 + 2n)   n+2
E da injetividade da função f temos:





   ( n   ) ln( ( n^2 + 2n + 1 ) . n+1 )   <
ln(1)=0   ( (n^2 + 2n   )n+2)
Isto é:

   (n)(n+1)ln( ( 1 + 1 )  ) – ln( ( 1 + 1 ) 
 )<0   ( (  n )  )( ( n+1  )  )






Logo,





n n+1( 1 + 1 ) < ( 1 + 1 )(   n ) ( n+1 )   
   C.Q.D
P.S.: Se tiver algum erro me avisem por favor.
From: esdrasmunizm...@gmail.com
Date: Thu, 28 Jan 2016 12:18:03 -0300
Subject: Re: [obm-l] Ajuda numa desigualdade.
To: obm-l@mat.puc-rio.br

L = ((1+1/(n+1))^(n+1))/(1+1/n)^n = ((1 - 1/(n+1)²)^n)((n+2)/(n+1)) 
Use que (1 - x)^n > 1 - nx, Para x \in (0, 1)
L > (1 - n/(n+1)²)((n+2)/(n+1)) = ((n²+n+1)/(n²+2n+1))((n+2)/(n+1)) = 
(n³+3n²+3n+2)/(n³+3n²+3n+1) > 1.


Esse último termo é maior que 1.
Em 28 de janeiro de 2016 09:41, Douglas Oliveira de Lima 
 escreveu:
Opa Marcelo, muito obrigado mesmo, eu estou procurando uma solução daquelas 
tipodesigualdades, onde efetuamos uma estratégia para chegar no resultado, tipo 
uma daquelas que tu encontra no livro de combinatória do MOrgado(o problema das 
apostas).Mas valeu, se conseguir uma dessas me manda novamente por 
favor.AbraçoDouglas Oliveira
Em 28 de janeiro de 2016 01:26, Marcelo Salhab Brogliato  
escreveu:
Oi, Douglas, tudo bem?
Se provarmos que f(x) = (1 + 1/x)^x é estritamente crescente, então está 
provada sua desigualdade.
Uma maneira é fazer isso usando cálculo. Seja g(x) = ln(f(x)) = x ln(1 + 1/x). 
Assim, se provarmos que g(x) é estritamente crescente, então f(x) também será 
(exercício: prove essa afirmação).
g'(x) = ln(1 + 1/x) + x * (-1/x^2) / (1 + 1/x)  = ln(1 + 1/x) - (1/x) / (1 + 
1/x) = ln(1 + 1/x) - 1/(1+x)
Temos que mostrar que g'(x) > 0 para todo x.
Sabemos que ln(x) < x - 1, para x != 1. Aplicando essa desigualdade em 1/x, 
temos: ln(1/x) < 1/x - 1 => ln(x) > 1 - 1/x, para x != 1.
Aplicando a desigualdade acima em 1+1/x, temos: ln(1+1/x) > 1 - 1/(1 + 1/x) = 
(1/x) / (1 + 1/x) = 1/(1+x). Logo: ln(1+1/x) > 1/(1+x) => g'(x) > 0 para todo x 
(já que 1+1/x > 1).
Abraços,Salhab
2016-01-28 0:34 GMT-02:00 Douglas Oliveira de Lima 
:
Olá caros amigos, gostaria de uma ajuda na seguinte desigualdade 
(1+1/n)^n<(1+1/n+1)^(n+1), para n natural.
Agradeço desde já.







-- 
Esdras Muniz Mota
Mestrando em Matemática
Universidade Federal do Ceará



  

RE: [obm-l] Ajuda numa desigualdade.

2016-01-28 Por tôpico Douglas Oliveira de Lima
Erro? Bom no meu celular acho que saiu as fórmulas todas fora de ordem rs
Em 28/01/2016 16:02, "Bruno Lira"  escreveu:

> Primeiramente, tome a função logaritmântica f(x) = ln(x) cujo
>
> domínio é o conjuntos dos números reais maiores que ou
>
> igual a zero. Note que a função f é injetora. Portanto,
>
> para provarmos que:
>
>
> n n+1
>
> ( 1 + *1* ) < ( 1 + * 1 * )
>
> (   n ) ( n+1 )
>
>
> basta provar que:
>
>
>(n)   (  n+1)
>
> ln( ( 1 + *1* ) ) < ln( ( 1 + *  1 *)  )
>
>( (   n ) )   ( ( n+1)  ) .
>
>
> De fato, temos que:
>
>
>(n)( n+1)
>
> ln( ( 1 + *1* )  ) – ln( ( 1 +*   1   *)   ) =
>
>( (  n )  )((  n + 1)   )
>
>
>(n)(n+1)
>
> ln( ( *n** + 1 *)  ) – ln( ( *n** + **2* ) ) =
>
>( (n) )( ( n + 1 ))
>
>
>(   2n  )
>
> ln( (*  n + 1  *) . *n+1* ) ; Das propriedades de logaritmo.
>
>( (n (n+2))   n+2 )
>
>
> Daí:
>
>
>( n   )
>
> ln( ( *n^2 + 2**n** + 1 *) . *n+1* )
>
>( (n^2 + 2n   )n+2)
>
>
> Como n^2 + 2n < n^2 + 2n + 1 e n+1 < n + 2 temos que:
>
>
> n
>
> ( *n^2 + 2**n** + 1 *) . *n+1*<1
>
> (n^2 + 2n)   n+2
>
>
> E da injetividade da função f temos:
>
>
>( n   )
>
> ln( ( *n^2 + 2**n** + 1 *) . *n+1* )   
>( (n^2 + 2n   )n+2)
>
>
> Isto é:
>
>
>(n)(n+1)
>
> ln( ( 1 + *1* )  ) – ln( ( 1 + *1 *)  )<0
>
>( (  n )  )( ( n+1  )  )
>
>
> Logo,
>
>
> n n+1
>
> ( 1 + *1* ) < ( 1 + *1 *)
>
> (   n ) ( n+1 )
>
>
> C.Q.D
>
> P.S.: Se tiver algum erro me avisem por favor.
> --
> From: esdrasmunizm...@gmail.com
> Date: Thu, 28 Jan 2016 12:18:03 -0300
> Subject: Re: [obm-l] Ajuda numa desigualdade.
> To: obm-l@mat.puc-rio.br
>
> L = ((1+1/(n+1))^(n+1))/(1+1/n)^n = ((1 - 1/(n+1)²)^n)((n+2)/(n+1))
>
> Use que (1 - x)^n > 1 - nx, Para x \in (0, 1)
>
> L > (1 - n/(n+1)²)((n+2)/(n+1)) = ((n²+n+1)/(n²+2n+1))((n+2)/(n+1))
> = (n³+3n²+3n+2)/(n³+3n²+3n+1) > 1.
>
>
>
> Esse último termo é maior que 1.
>
> Em 28 de janeiro de 2016 09:41, Douglas Oliveira de Lima <
> profdouglaso.del...@gmail.com> escreveu:
>
> Opa Marcelo, muito obrigado mesmo, eu estou procurando uma solução
> daquelas tipo
> desigualdades, onde efetuamos uma estratégia para chegar no resultado,
> tipo uma daquelas que tu encontra no livro de combinatória do MOrgado(o
> problema das apostas).
> Mas valeu, se conseguir uma dessas me manda novamente por favor.
> Abraço
> Douglas Oliveira
>
> Em 28 de janeiro de 2016 01:26, Marcelo Salhab Brogliato <
> msbro...@gmail.com> escreveu:
>
> Oi, Douglas, tudo bem?
>
> Se provarmos que f(x) = (1 + 1/x)^x é estritamente crescente, então está
> provada sua desigualdade.
>
> Uma maneira é fazer isso usando cálculo. Seja g(x) = ln(f(x)) = x ln(1 +
> 1/x). Assim, se provarmos que g(x) é estritamente crescente, então f(x)
> também será (exercício: prove essa afirmação).
>
> g'(x) = ln(1 + 1/x) + x * (-1/x^2) / (1 + 1/x)  = ln(1 + 1/x) - (1/x) / (1
> + 1/x) = ln(1 + 1/x) - 1/(1+x)
>
> Temos que mostrar que g'(x) > 0 para todo x.
>
> Sabemos que ln(x) < x - 1, para x != 1. Aplicando essa desigualdade em
> 1/x, temos: ln(1/x) < 1/x - 1 => ln(x) > 1 - 1/x, para x != 1.
>
> Aplicando a desigualdade acima em 1+1/x, temos: ln(1+1/x) > 1 - 1/(1 +
> 1/x) = (1/x) / (1 + 1/x) = 1/(1+x). Logo: ln(1+1/x) > 1/(1+x) => g'(x) > 0
> para todo x (já que 1+1/x > 1).
>
> Abraços,
> Salhab
>
> 2016-01-28 0:34 GMT-02:00 Douglas Oliveira de Lima <
> profdouglaso.del...@gmail.com>:
>
> Olá caros amigos, gostaria de uma ajuda na seguinte desigualdade
> (1+1/n)^n<(1+1/n+1)^(n+1), para n natural.
>
> Agradeço desde já.
>
>
>
>
>
>
> --
> Esdras Muniz Mota
> Mestrando em Matemática
> Universidade Federal do Ceará
>
>
>


[obm-l] Dúvida função

2016-01-28 Por tôpico Israel Meireles Chrisostomo
Olá pessoal eu gostaria de provar que uma função admite máximo sem calcular
o máximo da função, isto é possível?
Por exemplo, seja f(a,b,c) uma função, eu quero provar que a,b,c admite
máximo sem calcular seu máximo, lembrando f(a,b,c) é uma função de 3
variáveis, alguém por favor poderia me ajudar?


[obm-l] Função de 3 variáveis

2016-01-28 Por tôpico Israel Meireles Chrisostomo
Alguém tem um argumento (sem usar cálculo), para afirmar que a função
ab+bc+ac-2abc admite máximo (não precisa calcular o máximo)para qualquer
valor de k tal que a+b+c=k?